LSAT and Law School Admissions Forum

Get expert LSAT preparation and law school admissions advice from PowerScore Test Preparation.

User avatar
 Dave Killoran
PowerScore Staff
  • PowerScore Staff
  • Posts: 5850
  • Joined: Mar 25, 2011
|
#44104
Complete Question Explanation
(The complete setup for this game can be found here: lsat/viewtopic.php?t=8548)

The correct answer choice is (D)

In attacking this question, keep in mind that since each answer choice is supposed to be a complete and accurate list of the birds not in the forest, all of the birds not named on each list will be in the forest. Given the number of negative grouping rules in play, that is an important consideration. For example, in answer choice (B), when only H and G are not in the forest, J, M, S, and W are in the forest. But, according to the fourth inference, J and W cannot both be in the forest and therefore answer choice (B) is incorrect. Answer choice (E) can be eliminated by identical reasoning. Answer choice (C) can be eliminated because, via the third inference, H and W cannot both be in the forest. Answer choice (A) can be eliminated by applying the last rule. Answer choice (D) is therefore proven correct by process of elimination.
 ellesat
  • Posts: 3
  • Joined: Jan 29, 2013
|
#7424
I really appreciate you creating this forum and providing so much guidance. I have many more pertinent questions, but I'll start with this since I am on pg. 249. For the answer to Q. 6 the book states that B is incorrect because when H and G are not in the forest, J, M S and W are in the forest, and J and W cannot happen together. But how is W there? when g is not in the forest isn't W also not per the contrapositive of W-> G? Per this reasoning i still decided b to be incorrect, thinking h and g would not be an accurate list of missing birds because W also couldnt be there. What rule am I forgetting here ? Is it because Not H-> Not J, and Not J->....? Thanks.
Last edited by ellesat on Tue Jan 29, 2013 9:35 pm, edited 2 times in total.
 ellesat
  • Posts: 3
  • Joined: Jan 29, 2013
|
#7426
W is there because the answer choice is stating that h and g can be the only ones missing despite any rules of how the lack of G affects W. So B is also wrong because m and w could not both be there. Correct?
User avatar
 Dave Killoran
PowerScore Staff
  • PowerScore Staff
  • Posts: 5850
  • Joined: Mar 25, 2011
|
#7431
Hi Elle,

Glad we can help!

In question #6, answer choice (B) is an example of an answer that has a lot of problems. In List questions, having any single reason to knock out an answer is sufficient to kill it off, and thus once we see anything--in this case the JW inference--I eliminate (B) and do not address additional reasons as to why it is dead. That said, yes, if G is not in the forest, then via the contrapositive W is also not in the forest, which kills this answer. Another reason is from the application of the second rule where if M is in the forest then H must also be in the forest. Any of these would work. As they say, there is more than one path to the top of the mountain :-D

Please let me know if that helps. Thanks!
 ellesat
  • Posts: 3
  • Joined: Jan 29, 2013
|
#7440
Hi Dave,
Yes, I recall you stating in earlier chapters that any reason is enough to eliminate the answer. I was confused by the wording of the question but your response helped a lot. Thanks!
 Xantippe
  • Posts: 14
  • Joined: Nov 18, 2014
|
#67880
The Q stem reads; Which one of the following could be a complete and accurate list of the birds NOT in the forest? The answer is (D) grosbeaks, martins, shrikes, wrens.

I need confirmation, for why m is in the out group.

Is it because of the m-->h and its' contrapositive not h--> not m relationship? The contrapositive places not m as the necessary, so it could be in the out group?



Thanks in advance!
 James Finch
PowerScore Staff
  • PowerScore Staff
  • Posts: 943
  • Joined: Sep 06, 2017
|
#67901
Hi Xantippe,

Because J or M :arrow: H, the only thing that would affect our knowledge of M is if we know that H is out, forcing both J and M out as well. If H is in, as it is in all answer choices except (B), this tells us nothing about J or M, either one of which or both could be out. J being out forces S in, but we don't know anything from having M out.

Hope this helps!
 leslie7
  • Posts: 73
  • Joined: Oct 06, 2020
|
#83554
Hi, I think the way the question was tackled was great. Figuring out the answers was based off of what was not listed in question 6 e.g. J <-/->W for answer choice B.

I struggled because I focused on what was listed , trying to track all the "nots" and whether the "nots" could exist together...

Would you suggest that in a list question such as this to make it a practice to focus on what they have not listed instead? Is that a more strategic/effective way more generally based off your experiences?
 Adam Tyson
PowerScore Staff
  • PowerScore Staff
  • Posts: 5153
  • Joined: Apr 14, 2011
|
#83982
In a game with just two groups, which includes this sort of "In/Out" game, that's one very good strategy to use, leslie7. That's the trick to these questions - you have to think about what is missing as much as what is present.

But another way to handle this is through the conditional chains created by the game. This chain results from putting all the rules together:

S :arrow: J :arrow: H :arrow: G :arrow: W (M is something of a random in this case - it could be in or out)

and the contrapositive:

W :arrow: G :arrow: H :arrow: M & J :arrow: S (everything accounted for, M is no longer random)

So, can the "out" group be just J and S? No way - as soon as S is out, J is in (first chain)

Can H and G be the only ones out? Nope - H out forces J and M out (second chain) and G out forces W out (first chain)

Can GJM be the out group? No, because G out again forces W out (first chain)

How about GMSW out? That works in the first chain! Winner

What about M and S out? The first chain is triggered when S is out, so G and W have to also be out, so no.

If the game can be diagrammed with a single, powerful conditional chain and its contrapositive, that can be an easy way to read through the answers, eliminating any that violate some portion of those conditional relationships.

Get the most out of your LSAT Prep Plus subscription.

Analyze and track your performance with our Testing and Analytics Package.